LSAT PT 04 Expl Unlocked
LSAT PT 04 Expl Unlocked
PrepTest 4
Explained
a guide to the february, 1992 exam
2006 Kaplan, Inc.
All rights reserved. No part of this book may be reproduced in any form, by photostat, microfilm, xerography, or any other
means, or incorporated into any information retrieval system, electronic or mechanical, without the written permission of
Kaplan, Inc. LSAT is a registered trademark of the Law School Admission Council.
Section I: Logical Reasoning
1
PrepTest 4 Explained
argument at all. Of course, since none of this is and the argument flows smoothly; that is, the
contained in the argument, this is all conjecture on our conclusion follows perfectly from the evidence if (C) is
part, which means we must reject (C) immediately as added to the mix. Deny (C) and the argument falls
irrelevant. apart.
(D) The event (D) describes—increased consumption (A) According to the stimulus borrowers can’t be
—happens after the price has already dropped. The leaders, but for that to be true, we don’t have to
relevant issue, however, is how much prices will drop; assume that leaders have to be lenders; a leader might
what happens afterwards is besides the point. follow Polonius’ advice and neither a borrower nor a
lender be.
4. (D) (B) confuses what is necessary with what is sufficient.
A survey of ferry passengers came up with startling Just because it is necessary that world leaders do not
results: Symptoms of seasickness were more common have the terms of their dealing set by other nations,
among those who took anti-seasickness medication the argument needn’t assume that setting the terms of
than among those who didn’t. Based on this, the dealings with other nations is sufficient for a nation to
author concludes that people would be better off not be a world leader.
taking anti-seasickness drugs. (D), holding that the (D) actually contradicts the stimulus, which says that
people who took the drugs would have experienced lender nations set the terms of dealings with borrower
even harsher symptoms without them, weakens this nations; stated that way means this is always the
conclusion by showing that anti-seasickness drugs case—no exceptions. No choice that contradicts the
actually do some good. stimulus can be an assumption.
(A) First, there’s no indication that the weather was (E) certainly sounds reasonable, but that doesn’t mean
rough on this passage. Second, (A) does nothing to it’s assumed by the argument. If you deny (E), and say
explain the curious fact that those who took the that such an isolationist nation can be a world leader,
medication were dispropor tionately afflicted with you don’t hurt the conclusion that a borrower nation
seasickness. can’t be a world leader.
(B) is simply irrelevant. It doesn’t matter who conducted
the tests; what’s important are the results. (One may 6. (E)
argue that who conducted the tests is relevant in some Rotelle claims that Sims is too old to address the
way to the results, but one would have to take this important concerns that currently face the country.
reasoning a number of steps further to actually show Sims’ reply is that she doesn’t want to make age an
how this weakens the stated conclusion.) issue, so she won’t comment on Rotelle’s youth and
(C) works as a mild strengthener; it suggests that the lack of experience. How does she counter Rotelle’s
survey’s sample was representative, and therefore claim that she is too old? She sidesteps it; she never
supports the accuracy of the results gathered by the squarely tackles the issue of her own age, but instead
survey. brings up the issue of Sims’ age (while pretending not
(E) If people who spend money on anti-seasickness too). So (E) is correct—she fails to respond directly to
drugs are less likely to repor t symptoms of the claim that she’s too old to govern.
seasickness, then it’s probable that even more people (A) On the contrary: Sims doesn’t directly address
who took the drugs suffered from seasickness than we Rotelle’s claim, so she can’t be said to demonstrate
had previously thought. This can only strengthen the that Rotelle’s claim is incorrect.
argument. (B) falls for Sims’ little rhetorical trick. She claims that
she won’t make an issue of age, then goes ahead and
5. (C) contradicts herself by referring to Rotelle’s
“World leader” is an idea that only appears in the inexperience and youth; so she definitely mentions age.
conclusion; we need an assumption that connects that (C) Sims only says (hypocritically) that age shouldn’t
idea with the terms or ideas in the evidence of the first be an issue; she never advances any general method
sentence. (C) makes that connection: If a nation can’t for deciding which issues are important.
be a world leader when it has the terms of its dealings
(D) Sims doesn’t even reply to Rotelle’s claim, much
with another nation set by that other nation, then it’s
less show it to be contradictory; in fact, it’s Sims’ own
true that a borrower nation (because it has terms set
claim that is contradictory.
by the lender nation) can’t be a world leader. Add (C)
2
Section I: Logical Reasoning
3
PrepTest 4 Explained
(D) First of all, the acts of violence answer choice (D) 11. (C)
refers to may very well have been committed in war, The author concludes that current legislation regulating
which doesn’t count. More importantly, though, choice smoking on the premises of privately-owned businesses
(D) refers to violence increasing, whereas a weakener is an unjustifiable intrusion into the private sector. She
must suggest that documented interpersonal violence doesn’t deny that inhaling second-hand smoke is
didn’t increase. dangerous, but claims that this isn’t the most important
(E) As an account of France’s population in the consideration; the most important consideration, she
1300’s, (E) is ambiguous (first it increased then it says, is that the laws violate the right of private
decreased), so we don’t know how France’s population businesses to determine their own policies free of
in that century compared to France’s population in the government intervention. The principle in (C) lends
1200’s. Even so, it’s also unclear how the supposed credence to the author’s contention that the right of
population changes are connected to interpersonal business to be free from regulation should outweigh
violence. In essence, (E) tells us nothing. the government’s right to protect non-smokers. (C)
allows one to draw the conclusion that the smoking
10. (E) regulations aren’t justified.
Fixed nitrogen is an essential nutrient that normally (A) misunderstands the author’s objection; she never
has to be supplied to non-legumes like wheat by means denied that second-hand smoke is dangerous. (A)
of fertilizers. However, Rhizobium bacteria living on would imply that regulations to limit second-hand
legume roots produces fixed nitrogen. The author smoke may well be justified.
concludes that if we can develop wheat strains that (B) is an au contraire choice; it comes down in favor of
also allow Rhizobium bacteria to live on their roots, the regulations on smoking.
need for artificial fertilizers will decline. Why? The
(D) could only weaken the argument. (D) claims that
author must also believe (E), that Rhizobium growing
businesses have an obligation to protect against harm,
on wheat will also produce fixed nitrogen (just as it
which would undermine the conclusion. However, (D)
does when it grows on legumes), and the wheat’s need
applies only to employees in the workplace, which
for artificially-supplied fixed nitrogen will be reduced.
shifts the scope of the argument. No matter how we
But this is an assumption, since the evidence only says
slice it, (D) cannot help justify the conclusion.
that Rhizobium growing on legumes produces nitrogen.
If we negate or deny (E), and assert that Rhizobium (E)’s reasoning certainly follows a different path from
would NOT produce fixed-nitrogen on wheat, we’d then that of the author: (E) wants to find a compromise, but
have absolutely no reason to believe, as the conclusion the author comes down squarely on the side of
proposes, that the need for fertilizers would be reduced. business against government.
(A) introduces the concept of what biotechnology ought
12. (E)
to do, which is irrelevant; the conclusion only speaks of
what will happen if Rhizobium-friendly wheat is produced. We’re introduced to a foul-sounding substance called
“leachate,” which develops when water permeates a
(B) The conclusion merely says that the need for
landfill site. The most explicit thing we’re told about
artificial fertilizers will be reduced, not eliminated
leachate is that it escapes into the environment
entirely; it’s entirely possible that fertilizers will still be
whenever the landfill’s capacity to hold water is
needed to provide other nutrients.
exceeded, and it only escapes into the environment
(C) needn’t be assumed; even if some strain of grass when the landfill’s capacity to hold water is exceeded
already has Rhizobium bacteria living in its roots, and (that’s the meaning of the “if and only if” statement in
even if that Rhizobium produces fixed nitrogen (which the second sentence). We’re also told that a method to
(C) neglects to say), the conclusion that the overall dispose of leachate must be found; currently, most
need for chemical fertilizers would be reduced by the leachate is sent to sewage treatment plants, but some
production of further new strains of Rhizobium-friendly sewage treatment plants can’t handle leachate. We
wheat wouldn’t thereby be invalidated. need a good inference based on this mess, and it
(D) Nothing in the argument requires that no crops comes out of the “if and only if” statement. We know
other than legumes produce their own fixed nitrogen; if that if the landfill’s capacity to hold liquid is exceeded,
there were rutabagas or tomatoes, for example, that leachate is certain to escape into the environment,
also produced fixed nitrogen, it wouldn’t damage the which means that answer choice (E) is inferable: if
conclusion that Rhizobium could reduce wheat’s need leachate doesn’t escape into the environment, the
for artificial fertilizer. landfill’s capacity hasn’t been exceeded.
4
Section I: Logical Reasoning
(A) The stimulus does say that leachate generally 14. (C)
escapes into the environment in unpredictable Despite the fact that people suffering from acute W
quantities, and that not all sewage plants can deal with already have lower average blood-fat levels than the
leachate, but nowhere implies that the two facts are population at large, most doctors believe that reducing
related; there’s no reason to think the ability to predict blood-fat levels is a good way to prevent acute W. This
the escape of leachate would help with the disposal is an apparent paradox. The key lies in the relationship
problem. between low blood-fat levels and the disease. (C) says
(B) contradicts the passage, which says leachate only that a person can only contract acute W when the
escapes into the environment when a landfill’s ability agent that causes acute W absorbs large quantities of
to hold liquids is exceeded. fat from a person’s blood. This tells us two things:
(C) is unsupported; the stimulus says “not all” treatment First, a person must have large quantities of fat in his
plants can handle leachate, not that none of them can. bloodstream to contract acute W—this explains why
reducing blood-fat is a good way to avoid contracting
(D) The stimulus doesn’t say that any leachate is
acute W. Second, people can only contract acute W
actually sent to any of the plants that can’t handle it;
after the agent has already absorbed a lot of blood
only that such non-leachate-friendly plants do exist.
fat—this explains why people who actually have acute
W have low blood-fat levels. So (C) explains how both
13. (D)
statements in the stimulus can be true, and thus
The author says that academic libraries (used only by resolves the paradox.
academic researchers) will have to cut down their
(A) is out of the scope, dealing as it does with people
subscription lists; journals are just too expensive. He
who have already been cured of acute W; (A) says
suggests that subscription decisions in each discipline
nothing about why their blood-fat was low when they
should be based only on the usefulness of the journal
had acute W, or why people should reduce blood fat to
in that discipline, and goes on to say that a journal’s
keep from contracting the disease.
“usefulness” can be measured by how frequently it’s
cited by researchers in published writings. (D) weakens (B) contains a double scope shift: It points to a
this suggestion by attacking the author’s concept of “synthetic fat substitute” (not fat), and says that
“usefulness.” If, as (D) says, researchers will not animals who have been fed that substitute show
always cite a journal article that’s important in their “several symptoms” of acute W (not the disease itself).
work, then it’s possible that by using the suggested So the disease isn’t there, the fat isn’t there, and (B)
criterion for usefulness, some journals that are useful isn’t anywhere.
may not appear useful based only on frequency of (D) It’s hard to know what to make of (D); it doesn’t
citation. Thus the proposed response to the problem explain the correlation between low blood-fat and acute
would actually be counter-productive—many useful W, and certainly doesn’t explain how reducing blood-fat
journals could end up being removed from the academic could prevent acute W—these people already have
libraries. acute W.
(A) and (C) are irrelevant. The problem under discussion (E) is also out of the scope. The stimulus is only
is experienced by libraries that cater only to academic interested in prevention of acute W, not in other
researchers, and so the nonacademic readership of diseases. (E) doesn’t address the paradox at all.
scholarly journals, (A), and the conditions of
nonacademic libraries, (C), are out of the scope. 15. (D)
(B) offers an irrelevant comparison/distinction; there’s The key to this question is the phrase “in this respect.”
no necessary connection between the length of a We’re told that carbohydrates increase the brain’s level
journal article and the number of other articles it cites, of serotonin, and that “in this respect” carbohydrates
or is cited by. Furthermore, in the stimulus, the act on the brain in the same way as antidepressants.
comparison between journals is only made within each Critical reading tells us that the “respect” in question
discipline—there is no comparison between disciplines must be the increase in serotonin—the way the
as stated here. sentence is structured, the author simply can’t be
(E) doesn’t affect the solution. It doesn’t matter that referring to anything else. And that leads right to the
controversies spill over from one academic journal to correct inference in (D): Some antidepressants must
another; so long as all the articles concerned with the also boost the brain’s level of serotonin.
controversy cite their sources, the solution in the
argument isn’t weakened.
5
PrepTest 4 Explained
6
Section I: Logical Reasoning
(A) The author doesn’t challenge the claim’s relevance, 20. (B)
he attacks it as counterproductive. He says that Previewing the stem, we see that we’re looking for the
making the definition of intelligent life more precise argument-objection that functions most similarly to
would actually have negative results; that’s not the Lee’s objection to Pamela’s argument. Pamela thinks
same as saying that the claim is irrelevant. businesses should help employees take care of their
(B) The author doesn’t offer any examples; after all, children—the future customers and employees—as
his argument concerns types of intelligent life that we this will benefit businesses in the long run. Lee’s
can’t imagine, so he can’t give examples. objection is that no given company will have as its
(C) The author doesn’t argue that any adequate patrons and employees only the children of its own
definition of intelligent life is impossible. He merely employees; thus, it wouldn’t be to a company’s benefit
argues that to make the definition “more precise” to provide parenting services when other companies
would make it more difficult to recognize new types of don’t. Abstracting this into general form, Lee basically
intelligent life. (C) is a distortion of that argument. says that an individual will not receive any benefit from
his or her action unless others also perform the action.
(E) The author doesn’t attack the antecedent claim’s
The objection in (B) fits this mold: Air breathed by each
evidence; he doesn’t focus on the claim’s support, but
individual has been polluted by others, so it doesn’t
instead talks about its consequences.
make sense to act individually against pollution. As in
the stimulus, unless everyone pitches in, the individual
19. (D)
won’t benefit from his or her action.
The interiors of unsalted foods that are microwaved get
(A) The first speaker says that some action (building
hot enough to kill the bacteria that causes food
new roads) won’t be of any use, which is already a
poisoning, whereas the interiors of salted foods don’t
departure from the stimulus model; the reply is that
get so hot and are therefore more vulnerable to
the failure to take action would be even worse. Clearly
harmful bacteria that could lead to food poisoning. This
not parallel.
isn’t a difficult situation to imagine encountering in real
life. Armed with this knowledge, any reasonable person (C) is way off; it completely lacks the contrast between
would feel safer from food poisoning by not adding salt individuals performing an action and people in general
to food before microwaving. This notion is echoed in performing the action, which was the crux of Lee’s
correct choice (D). reply to Pamela.
(A) offers a comparison that’s impossible to verify (D) does contain a disagreement over the results of a
from the stimulus. The stimulus never even mentions particular course of action (lying), but in both the
exterior bacteria, whereas it does state for a fact that original statement and the objection, everyone is
there is harmful bacteria on the interior of food. performing the action.
There’s simply no basis in the passage for comparing (E) contains a general statement about human social
how much bacteria is on the exterior of food to how institutions, and an attack on the logic behind that
much is on the interior. statement (an attack on the idea that the past is a
(B) needn’t be true, because the author isn’t good predictor of the future). Again, no contrast
condemning microwaves themselves, just pointing out between everyone doing something and a few people
a risk in the use of microwaves with salted foods. For doing something.
all we know, conventional ovens may do an even worse
job of killing bacteria. 21. (B)
(C) Scope shift: The stimulus deals with the effects of Pedro draws a contrast between cloth diapers and
adding salt to food before microwaving, while this disposable diapers: disposable diapers are a threat to
choice is about adding salt after microwaving. Be the environment while cloth diapers aren’t. For
careful! evidence, he describes the bad environmental effects
of disposable diapers, without examining the effects of
(E) is unwarranted; we can’t conclude salt is the
using cloth diapers. He draws his conclusion (that
primary cause of these problems because other
people should use cloth diapers) by only looking at one
elements may do an even better job of protecting
side of the story. Maria attacks this shortcoming by
bacteria from microwaves.
pointing out some environmental drawbacks to the use
of cloth diapers. As (B) says, she indicates that
Pedro’s conclusion may be premature, given that he
hasn’t considered all the evidence about cloth diapers.
7
PrepTest 4 Explained
(A) Never does Maria argue with the evidence Pedro 23. (C)
cites against using disposable diapers—she simply What’s wrong with this picture? The problem is this:
points out that there’s also evidence against using The argument doesn’t take into account the
cloth diapers. composition of the random group tested. For example,
(C) couldn’t be farther from Maria’s method of argument let’s assume a random sample of 100 people, all non-
—she never even mentions the word “disposable” in cocaine users. Can we do this? Sure: no restrictions
her rebuttal to Pedro. are placed on the “random sample,” so why not use a
(D) is trickier, but upon close scrutiny you’ll notice that convenient one to test the argument’s claim?
it centers on a comparison that is never made. Maria According to the author, roughly five percent of this
does point out that there are drawbacks to the use of 100 people—that is, five people—will test positive,
cloth diapers, drawbacks Pedro hasn’t considered, but even though not one of the five is a cocaine user. Does
she never argues that cloth diapers are worse than the conclusion make sense in the context of this
disposables; she only points out that the issue isn’t as random sample? No—clearly, the vast majority, in fact,
black and white as Pedro thinks. all of those who tested positive, are NOT cocaine
users. The point is that the argument’s conclusion is
(E) is beyond the scope of the argument. Neither Maria
valid only if the random sample contains a high
nor Pedro discusses the economic advantages of
proportion of cocaine users; otherwise, it falls apart.
either type of diaper; their concern is the environment
Answer choice (C) correctly addresses this reasoning
alone.
error: The author fails to consider what proportion of
the population has used cocaine.
22. (B)
(A) Everything in this argument, from the evidence to
The upshot of this interesting experiment is that each
the conclusion, is numerical in nature. One would be
child associated the term “rolling pin” with only the
hard pressed to discern a value judgment from the
exact object he had previously encountered, not with a
text—it’s simply not there.
category of objects that all serve the same purpose.
So what can we infer from this? Since each boy only (B) It’s hard to figure out what (B) is getting at. The
identified the rolling pin he and his father had used, “properties” of the so-called average member or
and since all the rolling pins are different, we can infer society are never mentioned in the stimulus, and
(B): that no two boys associated the name “rolling pin” therefore couldn’t have been attributed to anyone.
with the same object. (D) Actually, the author’s statistics specifically admit
(A) There’s a subtle scope shift happening here; the that some cocaine users (roughly 1 out of 100, in fact)
children are asked to identify the pins, not explain how do not test positive. Whether the author ignores this or
they’re used. The mere fact that they couldn’t extend not, it plays no real part in the argument and is not the
the definition of rolling pin to other pins doesn’t mean source of the logical flaw.
they didn’t understand the function of the pin they did (E) is way out of the scope—the argument doesn’t
identify. advocate anything; the author isn’t promoting testing or
(C), if anything, runs counter to the stimulus, since the denouncing it, she’s simply giving the facts and coming
children did not seem to understand that the term to a conclusion, erroneous as it may be.
“rolling pin” can refer to a category of objects. If (C)
were true, chances are the boys would have been able 24. (E)
to identify the other rolling pins in the room. Stated algebraically, the argument breaks down as
(D) Mind the scope! The conclusion deals only with follows: “If X, then Y. Y, therefore X.” The only choice
rolling pins, so we have no right to extend this that does not exhibit this flaw is answer choice (E). If
conclusion to include all the utensils used in the a country is democratic, then the opinions of each of
experiment. its citizens must have a meaningful effect on its
government. So far, so good—“if X, then Y.” But look
(E) contradicts the stimulus. The children certainly
at the rest of the argument: “In none of these
could distinguish their own rolling pins from the others;
countries does each citizen’s opinion have an effect
otherwise, how could each child pick out from the rest
(NOT Y), therefore, Western countries aren’t really
of the rolling pins only the one he and his father had
democratic (NOT X). This is a valid invocation of the
used?
contrapositive; in plainer language, it simply makes
logical sense. There is no reasoning error in answer
8
Section I: Logical Reasoning
9
PrepTest 4 Explained
10
Section II: Reading Comprehension
(A) is beyond the scope of the text. The passage never 4. (C)
mentions any international organization, either before It’s almost word-for-word: Choice (C) nicely paraphrases
the mid-20th century or today, with the power to the information found in lines 48–52.
censure nations for not taking care of their own
(A) “Increased political pressure?” Nah. According to
territorial waters.
the author, comprehensive laws governing activities on
(B), (D), (E) All of these choices have a similar problem: the sea will arise out of an “international consensus.”
They presume that nations were not masters in their
(B) plays on (and distorts) a detail from the wrong
own territorial waters under certain circumstances. But
paragraph—paragraph 2. The only real info we get on
the passage indicates that they controlled their own
“jurisdictional disputes” is the second paragraph’s
territorial waters under all circumstances.
description of the lack of them; “greater number of
jurisdictional disputes” should simply sound wrong
3. (A)
right off the bat.
The whole theme of the first two paragraphs is that the
(D) plays on (and distorts) a detail from the wrong
nations just didn’t seem to care much about regulating
paragraph—paragraph 3.
their territorial waters, and that the international scene
pretty much took care of itself. In other words, “low (E) is beyond the scope of the passage, which never
priority,” as stated in correct choice (A). If you were makes a distinction between larger and smaller
clear on the gist of the first two paragraphs, you nations.
shouldn’t have needed to go back to the passage to
pick up this point, but if you were still a little unsure, 5. (C)
it’s right there in black and white: The last sentence of We know countries didn’t care much about regulating
paragraph 1 says that, before the mid-20th century, territorial waters, but now we’re asked “Why?”. Well,
most nations didn’t bother to regulate activities in their it’s no secret—we’re told the reason in lines 12–15,
territorial waters. Paragraph 2 says that, during the and (C) nicely paraphrases the explanation.
same period, activities in international waters were (A), (B), (D) All of these choices reflect either beliefs
basically unregulated as well. Evidently, what went on about or the situation in international, not territorial,
in the oceans was not of much concern to the waters.
international community, which once again points to
(E) This choice would be a good common sense guess
“low priority.”
if the passage were missing, but since we have the
(B) In this wrong choice, the testmakers sneak in a passage right in front of us, we know this was not the
common Logical Reasoning tactic, the scope shift: reason for the lack of regulation in question.
Nothing in the passages suggests that nations were “Jurisdictional conflicts” is a concept relating to the
uninterested in their territorial waters—lack of interest open seas, not territorial waters, so this choice can’t
in regulating these waters doesn’t equate to an overall help explain anything.
lack of interest in them. Further, nothing in the
passage suggests that nations felt that international 6. (D)
waters held unlimited resources.
The passage is an historical analysis—it discusses
(C) Although nations had the authority to regulate matters before the mid-20th century as well as today—
activities in their coastal waters, few were actually and it is about a problem that requires international
interested in doing so. Moreover, nations were attention—the regulation of activities on the oceans.
precluded by law, not custom, from regulating activities
(A) The passage doesn’t mention dates or probe
in international waters.
specific events, so chronology
(D) Au contraire. Lines 21–24 indicate that a nation
isn’t the right term for the text. Besides, the word
had the right to control the behavior of its citizens (but
“crisis” is a bit strong: A problem isn’t the same as a
only its citizens) in international waters.
crisis.
(E) Au contraire. Before the mid-20th century, nations
(B) The passage certainly can’t be called a “legal
could not extend their territorial control beyond three
inquiry,” even though it does touch on legal issues.
miles. Only recently have some extended control
Furthermore, there’s nothing in the text about “abuse
beyond this point.
of existing laws…”
11
PrepTest 4 Explained
12
Section II: Reading Comprehension
7. (E)
Questions 7–13
This choice essentially captures the author’s opinion,
Topic and Scope: Species diversity; specifically, the which is stated very clearly right up front in paragraph 1,
history and consequences of species diversity. and reiterated for good measure again in paragraph 4.
(A) The author never states that the loss of species
Purpose and Main Idea: The author’s purpose is to trace
diversity is an “irreversible process.” Paragraph 2
the growth and decline of species diversity throughout
shows that the number of species rises and declines in
history, as well as to argue that declining diversity is
a cyclical fashion over time. The author does say that
harmful to humanity. His specific main idea is that
the extinction of a particular species represents an
humanity will suffer serious, though unmeasurable,
“irreplaceable” loss; but the loss of a single species is
consequences if it doesn’t take measures to halt the
distinct from the larger biodiversity process.
current decline of species diversity.
(B) distorts the comparison between biological wealth
Paragraph Structure: Paragraph 1 introduces the and material/cultural wealth. The author states that we
author’s opinion about the current “biodiversity crisis.” understand the latter while we take the former for
Paragraphs 2 and 3 provide historical background on granted. Even without this, the issue isn’t important
the topic: paragraph 2 recounts the history of species enough to be the main idea of the passage.
diversity before the rise of humanity, while paragraph 3 (C) The author never claims that the great variety of
discusses humanity’s negative effect on species life on Earth is attributable to past episodes of mass
diversity. extinction. He simply notes that these episodes are
In paragraph 4, the author reiterates his earlier point part of the history of the biodiversity process. In any
that there’s no way to tell precisely what effects loss case, just like (B), this choice omits the most
of species diversity will have, though it’s certain to be important theme of the passage—the author’s concern
harmful. He goes on to note, however, that there are over the biodiversity crisis.
certain to be existential costs—the loss of the earth’s (D) is too extreme. While the author suggests that
heritage—and more concrete costs in the area of food, today’s biodiversity crisis is a potential disaster, he
medicine, and commercial products. doesn’t imply that it could be worse than past episodes
of mass extinction.
The Big Picture:
• Note the classic structure of this ver y 8. (E)
straightforward science passage. The author’s Does this question remind you of anything? It should;
opinion appears in the first paragraph, and all of it’s just like a Parallel Reasoning question found in
the subsequent paragraphs simply ser ve to Logical Reasoning. According to lines 10–18, species
justify and strengthen that opinion. diversity increased rapidly at first, then held pretty
• Whenever any author has a definite opinion about much constant for a time, and then began a slow and
a topic, be sure that you’re clear about exactly steady climb later. Choice (E) features the same
what that opinion is—several of the questions are pattern of development.
likely to test whether you’ve picked up on it. (A) An immediate decline eliminates this choice
• This passage is an example of why it’s foolish to immediately.
panic at the sight of a passage merely because (B) The initial fluctuation eliminates this choice
it’s scientific in nature and seems to have a lot of quickly.
“big words.” Sure, “biodiversity,” “multicellular
animals,” not to mention the “Mesozoic, Paleozoic, (C) is eliminated because of its “cyclical” pattern.
Cenozoic,” eras may make the passage appear to (D) The decrease in the last part of this choice
be complex, but in fact, it’s really not. As noted contradicts the pattern in lines 10–18.
above, the structure is clear, and the author’s
opinion is crystal clear as well—these are the 9. (D)
things that determine a passage’s difficulty level, In lines 27–29, the author mentions that between 77
and this passage, despite some scientific jargon, and 96 percent of all marine animal species became
simply isn’t very difficult to understand. extinct during the Permian extinction episode. In lines
26–27, the author says that the Cretaceous extinction
13
PrepTest 4 Explained
episode was “minor” in comparison to the Permian. head from that passage. In any case, the sure-fire way
Thus, we can conclude that less than 77 percent of to get this point is simply to scan the passage for the
marine animal species became extinct during the choices you’re not sure of—chances are you were able
“Cretaceous crisis.” to recall from memory at least three, and possibly all
(A) Nothing in the passage suggests that it was the four of the wrong choices as things that were definitely
“second” worst extinction episode in history. mentioned in the passage.
(B) This can’t be—it was “minor” in comparison to the (A), (C) Hunting (A) and deforestation (C) are both
earlier Permian extinction episode. mentioned in paragraph 3.
(C) is beyond the scope of passage, which never (D), (E) The expansion of humanity (D) and human-
compares the Cretaceous crisis to the current engineered changes to the environment (E) are both
biodiversity crisis. mentioned in paragraph 1.
(E) Don’t be fooled because this extinction period was
12. (A)
marked by the disappearance of the dinosaurs; this is
only what makes it the “most famous” extinction Back to paragraph 4 for this one: In lines 45–51, the
episode. However, this doesn’t necessarily mean that author argues that humans take biological wealth for
the dinosaurs “comprised the great majority of species granted because they don’t understand its value. In
that perished during the crisis.” For all we know, many contrast, he suggests, because humans understand
more non-dinosaur species may have perished during the value of material and cultural wealth, these things
this crisis. aren’t taken for granted.
(B), (E) The distinction in paragraph 4 is between
10. (B) material/cultural wealth, on the one hand, and biological
In lines 38–42, the author refers to the careless wealth, on the other. The connections offered in these
introduction of the Nile Perch into Lake Victoria as an two choices distort and expand upon this distinction,
example of a “biogeographic disaster.” Moreover, the and are in no way suggested by the author.
example of the Nile Perch appears in a paragraph that (C) The author implies just the opposite—that
discusses humanity’s negative impact on biodiversity. preserving our biological wealth will enhance our
It follows that this case is presented as an example of material wealth and preserve our cultural heritage.
how human error, or lack of foresight, using (B)’s (D) serves up an unsupported comparison that isn’t
wording, resulted in biogeographic disaster. implied in the passage. The author doesn’t rank the
(A) The Nile Perch hasn’t become extinct. Rather, this importance of material and cultural wealth as opposed
species now threatens the existence of other marine to biological wealth (he seems to think that all are
species in Lake Victoria because of human important to humanity)—he merely contrasts our
carelessness. understanding and appreciation of each.
(C) The passage doesn’t say when the Nile Perch first
emerged as a distinct species. It could have been long 13. (C)
after the Permian extinction episode. This choice captures that author’s uncertain but
(D) No; the “material wealth” issue comes from an pessimistic appraisal (especially in lines 6–8) of the
entirely different paragraph—paragraph 4. biodiversity crisis.
(E) Au contraire. The action taken in regard to the Nile (A) Because the planet’s biota is “largely unstudied
Perch is cited as an example of an action that has and unappreciated” and therefore “usually taken for
helped to accelerate the loss of biodiversity. granted,” the author wouldn’t claim that species loss
would affect the material wealth of nations as
11. (B) immediately as their biological wealth. Indeed, he
might well argue that the effects of this loss on
Tricky little testmakers, aren’t they? Pollution is never
material wealth wouldn’t be felt for quite some time.
explicitly mentioned here as contributing to the
extinction of species. But pollution sounds familiar, (B) plays on a detail in paragraph 2 regarding the end
doesn’t it? Where did we see something about of the dinosaurs during the Cretaceous extinction
pollution? From the previous passage, that’s where. episode and the consequent rise of humanity. Easy as
Pollution was mentioned there in relation to human it may be to supply our own logical leap here, in fact,
activities in international waters. Maybe it stuck in your the author never suggests that the same thing will
14
Section II: Reading Comprehension
15
PrepTest 4 Explained
14. (D)
Questions 14–20
This choices encompasses the author’s topic, scope,
Topic and Scope: Scholarship about women in the and purpose.
French Revolution; specifically, recent scholarship (A) and (C) focus on details. The different phases of
about women’s role in the French Revolution. women’s participation in the revolution (A) is an issue
taken up in paragraph 2, while the alliance between the
Purpose and Main Idea: The author’s purpose is to
women’s movement and other political movements (C)
describe and evaluate recent scholarship about women
is an issue probed in paragraph 3.
in the French revolution; her specific main idea is that
this recent scholarship has finally given adequate (B) distorts information in paragraph 1, which notes
attention to the role of women in the French Revolution. that recent studies “signal a much-needed reassessment
of women’s participation” in the French Revolution.
Paragraph Structure: Paragraph 1 reveals the topic, The author also points out that earlier studies ignored
scope, and purpose of the passage. Paragraph 2 the role of women in the revolution.
discusses some of the historical findings of the recent (E) The author never claims that some studies seek to
scholarship, especially the notion that women’s evaluate the women’s movement while others seek
participation in the revolution can be divided into three only to describe it. She implies that all of the recent
distinct phases. Paragraph 3 discusses scholarly studies are both descriptive and evaluative in nature.
inquiries into the eventual downfall of the women’s
movement. Finally, paragraph 4 provides the author’s 15. (C)
assessment of the recent scholarship.
Lines 16–19 indicate that Godineau believes that literary
The Big Picture: pieces like Gouze’s had no “practical effect” on
political matters. “Little impact” in choice (C) is a nice
• This passage is an ideal place to begin work on paraphrase of this sentiment.
the section since topic, scope, and purpose are
all evident very early on. Moreover, the structure (A) and (D) are beyond the scope of the passage.
of the passage is very predictable. After the Nowhere does the text discuss what Godineau thinks
introduction in paragraph 1, the next two about the capacity of Gouze’s contemporaries to
paragraphs explore the content of the recent understand her tract (A). Likewise, the text never
scholarship, while the final paragraph provides discusses whether Godineau thinks that Gouze’s work
the author’s assessment of that scholarship. was the most compelling written by a women in the
years 1789–1792 (D).
• This passage illustrates the impor tance of
previewing the entire section before attacking any (B) Since Godineau believes that Gouze’s tract had no
of the passages. Sometimes the third or fourth “practical effect” on politics, she would not endorse
passage will be the easiest but you’d never know the notion that this piece contributed to the formation
that unless you looked at all of the passages of women’s political clubs.
before attacking any one of them. Never (E) is also beyond the scope of the passage, which
underestimate the importance of beginning the discusses only political writings up to mid-1792.
Reading Comp section on a high note—this will
do wonders for your confidence which will 16. (B)
hopefully spill over to the rest of the section. The Lines 19–24 state that the clubs began as
same can be said for blowing away the first logic philanthropic organizations, but later evolved into
games on the Games section; it will give you the political advocacy groups with an agenda that included
feeling “hey, I can do these, even on the real pushing for women’s inclusion in the military. Put
test.” Previewing both the RC and LG sections differently, they “eventually developed a purpose
before jumping in is the best way to increase your different from their original purpose.”
chances of a solid start. (A) Lines 25–26 say that the mass women’s
movement grew out of the famine of 1795.
(C) The clubs were founded for philanthropic reasons;
only later did they advocate a military role for women.
16
Section II: Reading Comprehension
(D) The passage doesn’t say what the original purpose (E) is also beyond the scope of the passage. All
of male political clubs was, so there’s no basis for paragraph 3 says is that Landes and Badinter think
concluding that women’s clubs counteracted this that the women’s movement had no choice but to align
purpose. with these political traditions. There’s nothing about
(E) is beyond the scope of the passage, which doesn’t whether the movement itself thought that these
say anything about women’s clubs in 1795. In fact, alliances were beneficial or harmful.
there’s nothing about the history of these clubs after
the end of 1792. 19. (E)
In the last sentence of the passage, the author
17. (D) reiterates the previously-mentioned conclusion of
Paragraph 1 makes the point that recent scholarship Landes and Badinter—that the women’s movement
about women in the French Revolution has broken new was suppressed as a result of its association with
ground on the subject. The remaining paragraphs are other political traditions. Hence, the word “cost” refers
devoted to describing and evaluating this scholarship. to that suppression.
In other words, paragraph 1 places this scholarship in (A) refers to Rousseauist political philosophy, not to
context. Landes and Badinter’s view of the downfall of the
(A) What argument? The author never makes an women’s movement.
argument of her own about women’s role in the French (B) is a point made at the end of paragraph 2—a point
Revolution; she simply reports on and assesses the that is unconnected to Landes and Badinter’s work.
work of others. (C) This answer choice doesn’t even go so far as to
(B) What challenges? The author distinguishes recent refer to women in the French Revolution.
works from earlier works, but she doesn’t ever (D) confuses cause and effect: The “cost” to women
comment on possible future challenges to recent was the collapse of their movement, not the reason for
works. that collapse.
(C) plays on (and distorts) a tiny detail in the paragraph.
(E) is beyond the scope of the passage, which deals 20. (B)
only with scholarship about women in the French The author introduces several new studies about
Revolution. The text doesn’t discuss scholarship about women in the French Revolution in paragraph 1;
French women in the 18th century in general. describes the contents of those studies in paragraphs
2 and 3; and then provides a positive assessment of
18. (A) them in paragraph 4.
In lines 41–54, we see that Landes and Badinter (A) The “political and intellectual traditions”
attribute the decline of the women’s movement partly mentioned in the passage are discussed only in
to its inability to utilize its own political discourse. paragraph 3, and the author doesn’t criticize them.
Adopting the traditional language and vocabularies of (C), (D) Both of these choices focus on details. The
the time “diminished the ability of the women’s only chronological sequence (C) in the passage
movement to resist suppression.” This strongly appears in paragraph 2, while the only comparison of
suggests that Landes and Badinter would agree women’s political activities in different time periods
wholeheartedly with (A), that the movement would (D) occurs at the end of that paragraph.
have been better off if the women developed their own
(E) The author’s purpose is to describe and assess
political lingo in support of their cause.
new research, not to reexamine “a long-held point of
(B), (C) Au contraire. In lines 49–54, Landes and view.”
Badinter link the downfall of the movement directly to
its political alliance with Jacobin men (B), while in lines
41–49, they assert that women had no choice but to
adopt the established political vocabulary of the day (C).
(D) is beyond the scope of the passage. There’s
nothing in paragraph 3 to suggest that Landes and
Badinter think that the movement would have survived
had it not been suppressed militarily.
17
PrepTest 4 Explained
21. (B)
Questions 21–27
In lines 21–22, the author calls Herbert’s approach to
Well isn’t France just the popular topic on this LSAT the analysis of French Impressionism “not
administration? First the Revolution, now French Art. persuasive,” which is simply another way of saying
As always, start with the basics: pinning down the topic “inadequate.” The rest of the passage explains why the
and scope. author thinks this.
(A) focuses on a detail in paragraph 3.
Topic and Scope: Art historians’ views of French
Impressionism; specifically, Herbert’s interpretation of (C) distorts the passage. The author claims that
French Impressionism. Herbert’s work hasn’t successfully placed Impressionism
in an historical context. That’s not the same as saying
Purpose and Main Idea: The author’s purpose is to that historical context is irrelevant to interpreting
describe and take issue with Herbert’s analysis of Impressionist works.
French Impressionism; the author’s specific main idea (D), too, distorts the passage. The “ideological conflict
is that Herbert’s attempt to set French Impressionism and change” alluded to in the passage concerns the
in a “sociocultural context” isn’t convincing. interpretation of Impressionist works, not the works
themselves.
Paragraph Structure: Paragraph 1 argues that criticism
of French Impressionism has lately centered on the (E) The author is critical of Herbert, so it’s not likely
alleged sociocultural implications of Impressionist that she would endorse the notion that future analyses
paintings rather than on their stylistic merits, and cites of Impressionism will have to take his work into
Herbert’s book as a classic example of this new account.
approach to critiquing Impressionism. In the last
sentence of this paragraph, the author dismisses 22. (C)
Herbert’s analysis as not “persuasive.” This choice nicely paraphrases lines 3–5.
Predictably, paragraphs 2 and 3 explain why, in the (A) The author mentions the substance of Rewald’s
author’s view, Herber t’s analysis is untenable. book, but never comments on its “objectivity.”
According to paragraph 2, Herbert’s definition of (B), (D) The author notes Herber t’s “somewhat
French Impressionism is off. And, according to eccentric” selection of painters, not Rewald’s.
paragraph 3, he himself undermines his own analysis
(E) paragraph 1 makes it clear that Rewald’s book
by acknowledging that Impressionist paintings don’t
preceded the “ideological debates” about Impressionism.
really reflect the realities of France in the
Impressionists’ day.
23. (C)
The Big Picture: Lines 23–25 state that Herber t changed the
• This passage is a classic “book review” passage boundaries of Impressionism in order to put it in its
in which the author critiques the views of “proper…context.”
somebody else. If you run into a passage like this (A) Rewald, not Herbert, emphasized “form and style.”
one on LSAT day—and there’s a good chance that (B) The fact that Herbert is associated with a school of
you will—many of the questions will test to see art criticism that condemns Impressionism’s failure to
whether you’ve grasped the nuances of the represent industrial life doesn’t mean that he
author’s perspective. personally harbored a bias in favor of industrial life.
• Always keep an eye out for sentences in which (D) The author accuses Herbert of “redrawing the
the author’s voice comes through—like the traditional boundaries of Impressionism” in a bizarre
sentence in lines 21–22 here. Not only do they manner, but doesn’t accuse him of incorporating all of
enlighten you about authorial purpose, but they nineteenth-century French painting in his definition of
also often help you to predict the direction in Impressionism. In fact, the author criticizes Rewald for
which the text is going to move. restricting his analysis to the 1860s and 1870s.
18
Section II: Reading Comprehension
26. (D)
Paragraph 3’s general thrust is that not too much
about society can be inferred from Impressionist
paintings because Impressionists consciously
distorted reality for stylistic reasons.
(A) and (C) touch on a distinction made by the author,
not by Impressionists themselves.
(B) This quote comes up in the context of an argument
that denies the validity of exclusively concentrating on
the substance of Impressionist paintings.
(E) If anything, this quote supports the claim that
Impressionists ignored certain subjects to concentrate
on others.
19
PrepTest 4 Explained
SECTION III: 7) And finally, from the “J,” draw a line down to an “H.”
LOGIC GAMES Key Deductions: The first thing to do is to check your
list of entities and make sure that they’re all included
in the sketch. In this case they are, so you can fully
Game 1: Lawyers’ Salaries depend on the master sketch (and redrawings of it, as
needed) to answer all the questions.
Questions 1–6
There are a few things that you should notice right from
The Action: A quick look at the rules tells you the start. Kohn has the largest salary, period. Either
immediately that this is a “free-floating” sequencing Hae or Nassar has the smallest salary. Here’s a
game. The rules place entities in relation to each other common mistake to avoid: Just because Lopez and
instead of assigning them to specific spots (as in “the Nassar are connected to a shorter “branch,” don’t
mime was ranked fifth”). You are asked to order nine assume that they necessarily make a higher salary than
partners in a law firm—Fox, Glassen, Hae, Inman, any of the entities in the other “branch.” They could
Jacoby, Kohn, Lopez, Malloy, and Nassar—in relation both make less than even Hae. As soon as you’ve taken
to each other based on their salaries. The Key Issues some time to make sure you understand the sketch,
will be: you should be all set to rack up some easy points.
1) What par tner can, must, or cannot have a The Final Visualization: And here is our neat, accessible
higher/lower salary than what other partner? sketch:
2) What partner can, must, or cannot have the same
salary as what other partner?
more
K
The Initial Setup: Often, the best way to visualize this I L
kind of “free-floating” sequencing game is vertically. On
the top of the page, write “more” and at the bottom F N
write “less.” This will serve to remind you that those
partners above have a larger salary than those under M
(this also makes logical sense). That and listing the
partners (their letters actually) off to the side are about G
all you can do before hitting the rules:
J
FGHIJKLMN more H
less
less The Big Picture:
• In a “free-floating” sequence game, a common
The Rules:
mistake is assuming relationships that aren’t
To give yourself an idea where to start, scan the rules explicitly stated. If a relationship between two
and try to spot a partner that is not stated as having a entities is not bound by a rule or deduction, don’t
lower salary than another partner. Kohn in Rule 1 isn’t assume it.
explicitly lower than anyone, so put a “K” at the top of • Time taken up front is especially important in this
your sketch. type of game, but a careless master sketch can
1) From the “K” draw two lines, one down to an “I” and turn a straightforward game like this into a
one down to an “L.” nightmare. If your master sketch is accurate, you
shouldn’t have to use your pencil much in the
2) From that “L” draw a line down to an “N.”
course of this kind of game, except for an
3) Now jump over to the other branch. From the “I,” occasional re-drawing when necessary.
draw a line down to an “F.” • Picture the lines between the entities as elastic—
4) From the “F,” you need a line down to an “M.” you can stretch L and N way down below H, or
5) From the “M,” draw a line down to a “G.” place them practically anywhere between K and H
(and L and N don’t necessarily have to be near
6) From the “G,” draw a line down to a “J.”
one another, either). This flexibility of the entities’
20
Section III: Logic Games
relationships to one another is the key element of (A) L is now above F but could be above or below I, and
the “free-floating” sequence game. N could be anywhere below L. Keep on looking.
(B) L is between K and I, but N could be anywhere from
The Questions: third to last.
(C) N is now above F, but L and N are not set in
1. (D)
relation to I.
There are two kinds of partners who cannot have the
(D) If N is above I, N must be third, following K and L.
third highest salary: Those who must have a higher
This leaves I through H four th through ninth,
salary and those who must have a lower salary. Who
respectively. All of the par tners’ rankings are
must have a greater salary? Only K, who has the
determined, so (D) is the answer.
greatest salary, but she’s not one of the choices. So
look for those entities that have three or more people (E) L and N could assume many positions in the
over them in the sketch (they could be at best fourth ranking above M and below K, so (E) is no help.
highest). The right branch is composed of L and N
below K. We noted above that L and N are flexible and 4. (D)
can fit anywhere into the left branch, so either of them If N has the same salary as one other partner, what
could have the third highest salary. Eliminate answer CAN’T be true? At first glance, it appears that there’s
choices (C) and (E). On the left branch, count down not much to do here except note the new info and
three people, and anyone after that could be the check the choices. However, you might have noticed
answer. M, G, J, and H all have three or more people that L couldn’t be the same or less than H because
over them, which means that any of them qualifies as there would be no one left for N to share salaries with.
a partner that cannot have the third highest salary. The If you recognized this, you could have just scanned the
testmakers happened to pick M, answer choice (D). choices and found that choice (D) is impossible for this
reason. If you didn’t see this, you would need to check
2. (C) each choice, stopping when you found the choice that
You can probably picture this scenario in your head, but must be false.
you may have also opted for a quick re-drawing of the As for the wrong choices, here are orderings that show
master sketch including the new information (just draw that each of them is possible:
an “=” sign between M and N): (A) and (B) K—L—(I=N)—F—M—G—J—H
(C) K—I—F—L—(M=N)—G—J—H
K
(E) K—I—F—L—M—G—(J=N)—H
I L
5. (C)
F
What are the minimum number of different salaries?
M=N Well, L and N can share salaries with two other
partners in the left branch. The entities in the left
G branch, however, are all separated explicitly by the
rules. So all you have to do is count them up (don’t
J forget to count K at the top). There are 7, which is
choice (C).
H
6. (D)
Now it’s simply a matter of counting the people that are
definitely below L. The new sketch clearly shows that N, Here we’re looking for G’s possible rankings, given the
M, G, J, and H are all below L, a total of 5 people, answer fact that no one shares salaries. First count the people
choice (C). I and F could be below L, but they could also who are explicitly placed above G. There are four, which
be above L. K, as always, is definitely above L. means that G can be fifth but no higher than fifth. Does
this eliminate any choices? No, since fifth appears in
3. (D) each answer choice, and no answer choice includes a
place higher than fifth. Who else could earn a higher
To determine all the salaries, you need to connect the salary than G? The only “wild cards” are L and N who
L—N branch of the sketch to the longer branch with all are very flexible. One, both, or neither could be above
of the entities’ locations definitely set. Not much to do G. That makes G’s possibilities fifth, sixth, and
but check the choices. seventh, choice (D).
21
PrepTest 4 Explained
Game 2: Illnesses and Symptoms Key Deductions: We made a bunch of deductions along
the way as we went through the rules above. But
Questions 7–11 there’s even another deduction to be made from
combining Rules 4 and 5 with what we already deduced
The Action: In this one we’re asked to match five about illness L: L has either two or three symptoms
illnesses—J, K, L, M, and N—with their symptoms: a (Rule 4), but N has at least one symptom, so L can’t
matching game. The Key Issues deal with the typical have all three symptoms (there would be none left for
matching concerns: N—Rule 5). L, therefore, must have exactly two
1) What symptoms does each illness have? symptoms, and N must have exactly one symptom.
Go back and change the “2 or 3” to just “2” over the L
2) What illnesses can, must, or cannot have the same
and write “1” over N.
symptoms as what other illnesses?
Three of the five illnesses (J, K, and M) are entirely
The Initial Setup: Either a grid or a list works well with filled, and you know the exact number of symptoms for
a matching game. You could use a 5 x 3 grid with the the other two illnesses, L and N. This is a ton of
illnesses—J, K, L, M, and N—across the top, and the information and should lead to some quick and easy
symptoms—f, h, and s—along the side. This way you’d points.
put a “√” when you match a symptom with an illness
and an “X” when you know that an illness definitely The Final Visualization: So, here’s what we have
doesn’t have that symptom. However, if you were heading into the questions (and it’s quite a lot):
keeping track of illnesses’ symptoms in real life, you’d f h s
probably just make a list of the illnesses across the
page and be ready to fill in the symptoms under each, at least 1 same =
like so:
f h s 2 1
J K < L M N
J K L M N
no f f h/s f
The Rules: h no h h
s no s s
1) Very concrete—put an “h” for headache and an “s”
for sneezing under the J.
The Big Picture:
2) Think first, don’t just write “J ≠ K.” Each illness has
at least one symptom, and Rule 1 just said that J has • Again we see how important it is to take the time
headache and sneezing; therefore K must have only up front to work out as much as we can about the
fever. Put an “f” for fever and “no h” and “no s” under setup. The more you know about a game before
K. Also put “no f” under J. hitting the questions, the better. Anyone armed
with the large amount of deductive information
3) L will have at least one (or both) of headache and described above should be able to blaze through
sneezing. Let’s write “at least 1 same” and draw the five questions of this game, saving precious
arrows between J and L. time for the other games in the section.
4) Again, think before you draw. You know that K has • In Logic Games, the numbers aspect is always
exactly one symptom, so L must have two or three. impor tant. In this game, for example,
Write “2 or 3” over L. remembering that each illness has at least one
5) L and N don’t have any symptoms in common. Write symptom leads to the Key Deductions. Always
an “≠” with arrows pointing to L and N to serve as a paraphrase the rules, asking yourself “what does
reminder. this mean?” and not “what does this say?”
6) Don’t just write “M > J.” Think through the rule first.
What did Rule 1 say about J? J has two symptoms, so
M must have all three. Write “f, h, s” under M.
22
Section III: Logic Games
The Questions: front, checking each choice is a viable option, but the
active way, when available, is always better. If two
7. (E) illnesses have all three symptoms between them, then
This question is a ten-seconder after all of our work Walter, who has only two symptoms, cannot have all
with the setup. Scan the choices against what we’ve the symptoms of both. J and K have all the symptoms
deduced. N has one symptom, not two as choice (E) between them, since J’s symptoms are headache and
has it. sneezing, and K’s symptom is fever. Unfortunately, J
and K isn’t a choice. However, L and N also must have
8. (C) all three symptoms between them, since L has two
Once you’ve decoded this question stem, it’s just a symptoms, N has one, and they have none in common.
matter of checking for the pair who could share exactly So Walter, suffering from exactly two symptoms,
the same symptoms; that is, the same number and cannot have all the symptoms of L and N, Answer
type. M is the only illness with all three symptoms so choice (E). On Test Day, you wouldn’t bother with the
axe any choice that includes M, answer choices (D) other choices. You’d just mark (E) and move on. But for
and (E). We also know the numbers of symptoms for the record, here’s what’s wrong with the other choices:
each of the illnesses; the only pairs that can possibly (A) No, J and L can share headache and sneezing, so
have the same number of symptoms are J and L (they axe (A).
each have two symptoms) or K and N (they each have (B) N’s one symptom could be headache or sneezing;
one). J and L isn’t a choice, but K and N is, answer this kills (B).
choice (C).
(C) We just saw in question 9 that L could share one
of its two symptoms with K. Walter could have these
9. (A)
two, so axe (C).
Make abstract information concrete. L’s symptoms
(D) N’s one symptom could be fever, just like K.
aren’t exactly the same as any other illness. L has two
symptoms and so does J, but for this question they
can’t share both of them. J has headache and
sneezing. Rule 3 tells us that L shares at least (and in
this question, exactly) one with J. So L’s other
symptom must be fever, choice (A).
10. (E)
Read carefully here. You may have marked (A) thinking
that it was a direct restatement of Rule 3. But the
question stem says “exactly” one symptom; Rule 3
states that J and L share at least one symptom, which
means they can share two. Knowing the number of
symptoms in each illness helps enormously here. It
may have occurred to you that M, with all three
symptoms, and either K or N, with exactly one
symptom, would have to share exactly one symptom.
Scanning the choices we see that M and N is the pair
the testmakers chose in (E). If you didn’t see this, you
could have answered this one fairly quickly by simply
checking the choices:
(A) See discussion above.
(B) No, J with two symptoms must share exactly two
with M.
(C) and (D) Both of these pairs of illnesses could share
one symptom, but they can also share none.
11. (E)
What combination of illnesses would force Harold to
have all three symptoms? With all the deductions up
23
PrepTest 4 Explained
Game 3: Street Cleaning 5) Make sure you interpret this correctly. For now write,
“8…4…6,” and we’ll come back to this rule under Key
Questions 12–17 Deductions.
6) 2, 5, and 8 are cleaned in the afternoon, so at the
The Action: This game requires us to order seven far end of the PM row, write “2, 5, and 8.”
streets—1, 2, 3, 4, 5, 6, 7, and 8—based on when
they’re cleaned—either the morning or afternoon of Key Deductions: Let’s now look closer at Rule 5.
one Monday through Friday work week. This is a Street 4 is set on Tuesday AM and 6 is cleaned after
sequencing game with a slight twist. The morning and 4. Rule 6 says that 2, 5, and 8 are cleaned in the
afternoon aspect means that you’ve got ten spaces in afternoon. Now, combine Rules 5 and 6, and the only
which to plug the streets. The Key Issues will be: afternoon slot that is available before 4 is Monday PM,
1) When is each street cleaned? so 8 must be cleaned Monday PM; write an 8 in
Monday’s PM square. And cross off 8 from the list of
2) What streets can, must, or cannot be cleaned
afternoon streets.
before or after what other streets?
It’s also a good idea to identify the “floaters,” those
3) What streets can, must, or cannot be cleaned in the
entities that aren’t bound by any rules. Here the
morning?
floaters are 1 and 3. They can go just about anywhere.
4) What streets can, must, or cannot be cleaned in the However, there is one thing about these two floaters
afternoon? that you may have noticed. 4, 7, and 8 are all set. 6
must be cleaned after 4 on Tuesday PM, Wednesday,
The Initial Setup: If you were keeping track of these
Thursday, or Friday. 2 and 5 must be cleaned in the
days, you’d probably make a little calendar. A calendar
afternoon. So what about Monday AM? The only
is just a type of gird, so put the days—M, T, W, Th, and
streets that can be cleaned on Monday AM are 1 and
F—across the top of a 5 X 2 grid, and along the side
3. They are the only entities free to be cleaned on
write AM and PM, for morning and afternoon. Since the
Monday (6 can’t) AM (2 and 5 can’t). Write “1/3” in
testmakers were kind enough to number the streets,
Monday’s AM square. This is a tricky bit of deductive
your job will simply be to write the numbers into the
thinking, but if you saw it, you’re that much ahead of
square depending on when each street is cleaned. If
the game.
you know that no streets can be cleaned on a certain
day and time, just put an “X” in that square. Remember The Final Visualization: So here’s the very helpful
to list the streets (in this case, the numbers) off to the sketch:
side:
1 2 3 4 5 6 7 8 1 2 3 4 5 6 7 8
4...6
M T W Th F M T W Th F
AM AM 1/3 4 7 X
PM PM 8 X 2, 5
The Rules:
The Big Picture:
1) No streets are cleaned on Friday morning, so put a
• It’s preferable to work a rule directly into the
big X in Friday’s AM square.
master sketch (like Rule 5 above). When that’s
2) Wednesday afternoon is out, so place another X in not possible, then it’s better to rewrite it than to
Wednesday’s PM square. simply underline the rule. You will remember it
3) Easy enough; write a 4 in Tuesday’s AM square, and more readily after shorthanding it in your own
cross it off the list of streets. form.
4) And now 7 is definitely set as well. Cross 7 off your • The scratchwork you choose to use is up to you.
list of streets and put it in Thursday’s AM square. You could just as easily have listed M through F
across the page with two dashes under each for
24
Section III: Logic Games
25
PrepTest 4 Explained
Game 4: Ski Chalets Key Deductions: There is something that you should
notice just from reviewing the master sketch. Because
Questions 18–24 of Rules 4 and 5, the only chalets that can be
connected with M are J and N (and whichever one is
The Action: This is a type of logic game that has been connected to M will be maxed out thanks to Rule 3).
absent from the more recent LSATs: a mapping game.
Although you may not see one of this type on your test, The Final Visualization: Here’s the master sketch:
the methods used to solve this particular type are the
same as those needed to solve other more “regular” row 1: J K L 2 MAX
types. You are asked to map a single path to each of 5 TOTAL
six chalets—J, K, L, M, N, and O. The Key Issue is:
row 2: M N O NONE CROSS
1) What chalets can, must, or cannot be directly
connected to what other chalets to form a single
The Big Picture:
continuous path?
• Rewriting a rule in your own words (when you
The Initial Setup: You’re given a wonderful sketch in can’t build it directly into your sketch) is the best
the opening paragraph, so by all means, use it. As you way to remember it.
work with the rules, build them right into the given • Not all rules are in indented form. Some very
sketch. Here’s a reproduction of the diagram that was important rules (like “single continuous path”
provided: above) are hidden in the opening paragraph. Be
on the lookout for them.
row 1: J K L
The Questions:
row 2: M N O
18. (E)
The Rules: Not much to do with a “could be true” question with no
new information except check the choices. Way down
There’s one very important rule stated in the opening
at (E) we see O connected to L and O connected to N.
paragraph, and you could have easily missed it. It’s
All we have to do is connect M and J to get an
stated that “a single continuous path…connects all of
acceptable single continuous path.
the chalets.” This means that one single line must
connect all the chalets with no backtracks (that’s one (A) through (D) all try to connect M with a chalet other
of the implications of the word “continuous.”) So a than J or N, not possible.
single path will wind around touching all the chalets—
very important. 19. (C)
1) The path is composed of exactly 5 segments, each Since N and K are connected, both are maxed out. M
of which runs between exactly two chalets. must always be connected to either J or N, so here M
must be connected to J and J is maxed out. The only
2) This is a basic loophole closer; no chalet is left out
chalet that is left to be connected to O is L. You have
of the path.
a single continuous path and everything is set, so it’s
3) Each chalet is connected to at most two other a pretty simple task to scan the choices looking for the
chalets. This rule hints at the importance of the pair that must be connected. It’s L and O, (C).
numbers aspect in the game. When any chalet is
(A), (B), (D), and (E) are all impossible.
connected to two others, then it’s maxed out. Write “2
MAX” by the provided sketch to serve as a visual
20. (D)
reminder.
J and K are now maxed out. By now you should be used
4) During the course of winding around to each chalet,
to looking for a chalet to hook up with M, and here it’s
the path cannot ever cross itself. Write “none cross.”
N. That’s two chalets connected to N, so N is maxed
5) These rules are very concrete, so build them right out. Just like in question 19, L must be connected to
into the given sketch. Draw a line between J and N, and O, choice (D).
draw another one between K and L.
(A), (B), (C), and (E) are all impossible.
26
Section III: Logic Games
22. (A)
We’ve seen this basic concept before. M can only be
connected to either J or N, so in order to force M to
connect with N, all we need to do is max out J. This is
accomplished by connecting J with either chalet K or
chalet O; the testmakers chose the former as choice (A).
(B) through (E) all leave open the possibility that M is
connected to J, not K.
23. (C)
This should have been obvious because of all the
previous work with M. The thinking goes like this: M
can only be connected to J or N. If M is connected to
both of them, J and N are both maxed out, leaving no
way to connect all the chalets with a single continuous
path. M can be connected to only one chalet (J OR N),
not two, which is choice (C). You could have checked
each choice, but that would have taken much longer. If
you did have to try out the choices, you could have
sped things up by using your previous work, like so:
(A) In question 21, we saw K connected to two chalets,
so axe (A).
(B) In question 19 (and 20), we saw L connected to
two chalets; eliminate (B).
(D) In question 19, N was connected to two chalets.
(E) In question 21, we saw that O had to be connected
to exactly two chalets.
24. (B)
Here we’re told that no direct opposites are connected,
which means no J to M, no K to N, and no L to O.
Where to now? We’re used to connecting M to either J
27
PrepTest 4 Explained
SECTION IV: (A) and (C) are irrelevant, since neither says anything
about the fuel requirements of the two trips.
LOGICAL REASONING
(B) We have no idea what size ship is needed for a
Phobos trip or a Mars trip, so (B) doesn’t help us
1. (B) understand why the two trips of equal flight times have
Sound too good to be true? In real life, our natural different fuel requirements.
skepticism would lead us to ask: “Okay, what’s the (D) Huh? This confusing mess is an irrelevant
catch?” And the question stem straight out tells us to comparison. It doesn’t matter in the least how far
be suspicious—we’re asked to weaken the conclusion Phobos is from Mars; the fact that they are relatively
that this tax reform is good for the low income folks. close to each other doesn’t help explain the difference
Well, everyone knows that if you save money in one in fuel requirements for a trip from Earth to each of
area but lose more money in another, the overall these destinations.
transaction is a bust. And that’s the result set in
motion by choice (B): A low-income taxpayer who saves 3. (C)
$100 to $300 a year, but has to pay roughly $40 per
month, or $480 a year, in additional rent, certainly There’s a very subtle scope shift in this one: We’re told
isn’t benefiting from the tax reform. Based on one that research involving international collaboration has
positive effect of tax reform, the author concludes that produced papers of greater influence than has non-
tax reform will have an overall positive effect on low- collaborative research; influence, we learn, is
income taxpayers. However, if (B) is true, the tax measured by the number of times a paper is cited by
reform no longer looks like such a good deal. subsequent papers. We can accept this definition; it
stands to reason that papers used more frequently
(A) could only strengthen the argument, since saving than others by later researchers can be said to be
some taxpayers the expense of hiring an accountant more “influential.” But then the author expands the
would make the new tax reform seem like a positive scope hoping we wouldn’t notice—she concludes that
development. However, (A) fails even as a strengthener, projects conducted by international research teams are
since the argument concerns the interests of low- therefore more important than projects conducted by
income taxpayers, whereas (A) would apply to all single researchers. The evidence is that a paper’s
taxpayers. influence can be measured by number of citations,
(C) So what? This choice in no way suggests that tax while the conclusion is about a paper’s “importance.”
reform isn’t in the interest of low-income taxpayers. It may sound like the same thing, but actually, it’s not.
(D) and (E) both strengthen the argument. Allowing How “important” a paper turns out to be covers a far
low-income taxpayers to take a deduction on child-care greater scope than the “influence” it has had on
expenses (D), or releasing them altogether from the subsequent research as measured by the number of
obligation to pay taxes (E) are certainly benefits to times it’s cited. For the conclusion about “importance”
those taxpayers. to hold, the author must be assuming (C); that
“importance,” like “influence,” is indicated by the
2. (E) number of citations a paper receives.
The question stem tells us to concentrate not on the (A) would weaken the argument by suggesting that the
conclusion, but on the seemingly unusual relationship number of subsequent citations may not be a reliable
posited in the evidence. That is, although the author indicator of a paper’s importance.
concludes that Phobos should be next, the testmakers (B) Even if it were impossible to tell whether any given
don’t really care about that; they’re simply looking for paper is the product of an international collaboration
a reason why, flight times being equal, the Phobos trip (the denial of (B)), it still might be true that
requires so much less fuel than does the Mars trip. international collaborations are of greater importance,
Well, if the flight times are equal, odds are there’s so (B) cannot be an assumption.
some other difference between the flights that (D) is outside the scope. We don’t know anything about
accounts for the difference in fuel requirements. (E) collaborative efforts of scientists from the same
gives us what we want: If lift-off for the return trip from country; the argument only contrasts international
Phobos would use much less fuel than lift-off from collaborations with single author papers.
Mars would, then the mystery of the difference in fuel
requirements is solved.
28
Section IV: Logical Reasoning
(E) Even if (E) is true, it doesn’t indicate that (A) The stimulus said that all good parents are good
international collaborative efforts are more important listeners, but that doesn’t mean, as (A) infers, that
than single-author projects; the argument makes no every parent who is a good listener must therefore be
connection between funding and importance. a good parent. The stimulus allowed the possibility that
a lot of parents who are good listeners nevertheless
4. (A) fail to be good parents.
We’re asked to find a flaw in a very specific part of the (B) could be true, but it need not be true. It’s possible
argument: the claim that socialized medicine is that some parents are good listeners but not good
technologically superior to private-sector medicine. parents; on the other hand, it’s also possible that (A)
Notice that the only evidence the author cites for this is right and all parents who are good listeners are also
claim is the low infant mortality rate in countries that good parents. We can’t be sure, so (B) cannot be
use socialized medicine. (A) breaks this connection inferred.
between lower infant mor tality and technological (C) We know that most parents who are generous are
superiority by providing an alternative explanation for good parents, so we can conclude that most parents
the lower infant mortality; namely, that the greater who are generous are good listeners. But we have no
access to medical care in countries with socialized reason to believe the opposite, that most parents who
medicine, not technological superiority, might be the are good listeners are generous, so we can’t infer (C).
reason for those countries’ lower infant mortality rate.
(E) Although we’re told that “most” generous parents
If this is the case, then private-sector care may actually
are good parents, and thus good listeners, and “some”
be technologically superior, yet still have a higher infant
self-centered parents are good parents, and thus good
mortality rate than countries with socialized medicine
listeners, we can’t compare the two groups and
due to the limited access to such care.
conclude (E). Maybe the set of self-centered parents is
(B) is outside the scope; we’re not talking about the much larger than the set of generous parents, which
socialist economies and their achievements, but about could easily make (E) false.
the alleged technological superiority of socialized
medicine. 6. (A)
(C) is a strengthener; it supports the claim that the low In order for Kyra to conclude that the public should be
infant mortality rate connected to socialized medicine told to cut down on fiber intake, she must be assuming
indicates technological superiority. that public fiber consumption is actually at the high
(D) We don’t need a list of countries whose medical levels that cause problems in absorbing minerals. (A)
systems are “socialized” or “private sector” in order to weakens her argument by attacking this assumption: If
draw a conclusion about the merits of the different the average adult consumption of dietary fiber is only
medical systems. 10 grams a day, then adults aren’t even getting enough
(E) The author doesn’t presuppose the desirability of fiber to meet their daily recommended intake, and,
socialized medical systems. She provides evidence contrary to Kyra’s argument, the public shouldn’t be
(greater accessibility and low infant mortality) that she warned to cut back on fiber. People seem to be getting
believes demonstrates that desirability. too little fiber, not too much.
(B) is irrelevant. Kyra hasn’t said anything about
5. (D) processed foods and their fiber content, so (B) doesn’t
The stimulus is a set of formal statements about good weaken her claim; her conclusion concerns total fiber
parents. The most powerful statement is the one that intake in general, not its sources.
states that all good parents are good listeners. That (C) commits the “can vs. should” scope shift. The
means anyone who is a good parent is a good listener. issue in this argument is the proper level of fiber
So when the stimulus tells us that most parents who intake. The availability and/or affordability of that fiber
are generous are good parents, and some self- is another question entirely.
centered parents are good parents, it’s also telling us (D) functions more as a strengthener than a weakener.
that most parents who are generous are good One of Kyra’s worries is that calcium won’t be
listeners, and some self-centered parents are good absorbed if people eat too much fiber; (D) explains one
listeners. The last statement is just choice (D): if there of the reasons that calcium is important to good
are some self-centered parents who are good listeners, health. But you could have dismissed this choice right
that’s the same as saying that there are some parents
who are both self-centered and good listeners.
29
PrepTest 4 Explained
off the bat because it makes no mention of the discomfort is the main reason businessmen don’t run
argument’s most important element, dietary fiber. for president is seriously weakened.
(E) is also irrelevant to the main issue, which is the (A) The author never said that only businessmen act
amount of fiber in the average diet. Fiber’s “popularity” as fund-raisers or strategists, or even that most people
has no bearing on Kyra’s recommendation that fiber who act in such capacity are businessmen, so (A)
intake should be decreased for health reasons. wouldn’t even affect that part of the argument, no less
weaken the explanation in question.
7. (D) (C) Could be, but so what? (C) offers an irrelevant
We’re asked for an inference. Fortunately, the credited comparison while ignoring the crucial character in the
response is merely a consequence of the first explanation in question: businessmen.
statement in the policy paragraph: Every month, there (D) is irrelevant. The argument concerns the previous
is either a “holiday sale” or a “manager’s sale” going activities of those who seek to become president. The
on, or both. This means if there is no manager’s sale activities of former presidents after they leave office is
being run in some month, then there must be a holiday another matter entirely.
sale in that month, which is choice (D).
(E) The stimulus admits that businessmen have acted
(A) The stimulus only said that if there’s a holiday in a as fund-raisers and strategists in presidential
given month and there’s excess merchandise in the campaigns, so (E) fits in nicely; besides, the stimulus
warehouse, a holiday sale is declared. We can’t drew its conclusion about the character of individual
assume that only one of these conditions guarantees a businessmen, so the actions of companies are
holiday sale. irrelevant.
(B) needn’t be true; other months might lack a holiday,
or in some month the warehouse might not contain 9. (B)
excess merchandise. In that case we could just have a According to the definition, there are two requirements
manager’s sale. for a scientific theory to be “good.” It must be able to
(C) Nothing in the stimulus says that it’s impossible describe a broad set of events using only a few
for a manager’s sale to run when there’s excess elements, and it must make clear predictions
merchandise in the warehouse. Even if the excess concerning future events. Aristotle’s cosmological
merchandise gives rise to a holiday sale, we’re theory, which claimed that everything was made out of
explicitly told that it’s possible for both kinds of sales four elements, but failed to make any definite
to run at once. predictions, satisfied the first requirement but failed
(E) Although we know there’s no excess merchandise the second. For this reason, the author concludes that
in August, we don’t know that ONLY in August is there Aristotle’s cosmological theory was not a good one. All
no excess merchandise in the warehouse. the choices may be inferred from the passage except
(B). We don’t know anything about the “major concerns”
8. (B) of Aristotle’s theory or whether “the observation of
physical phenomena” is one of those concerns. (B)
The stem tells us what issue is paramount here: why
shifts the scope of the argument and cannot be
business executives don’t run for president. What’s
inferred.
the author’s explanation? Businessmen are used to
the hierarchical world of business and are (A) simply paraphrases the stimulus’s second condition
uncomfortable with the compromises and power- for a good scientific theory, that it must make definite
sharing that are necessary in politics; the skills and predictions about the results of future observations.
personalities that bring success in business don’t So if the stimulus statements are true, (A) necessarily
necessarily do the same in politics. But we’re also told follows.
what types of people do typically run for president: (C) The author said that Aristotle’s theory (which was
lawyers, military leaders, and career politicians. We based on four elements) fulfilled the first requirement
can weaken the author’s explanation if we show that of a good scientific theory, the simplicity requirement;
any of these ambitious types share the same attitudes therefore, a theory based on four elements must be
towards compromise and power-sharing as able to fulfill the simplicity requirement.
businessmen. (B) does just that: Military leaders are (D) The stimulus requires that a good scientific theory
just as uncomfor table with power-sharing and must account for many observations with a model that
compromise as businessmen. Since that discomfort contains few elements, so a theory that contains many
doesn’t deter military men, the explanation that such elements doesn’t make the grade.
30
Section IV: Logical Reasoning
31
PrepTest 4 Explained
a sample (his palace guards) that is likely to be (E) correctly identifies one of the problems (only one
unrepresentative. strategy has a hope of success), but suggests that
(A) The evidence (documents showing the loyalty of circumstances should be altered to fit that strategy.
the guard) doesn’t contradict the conclusion that Here, however, circumstances (the disease) are
Akhenaten’s subjects were loyal, it just fails to completely outside the physician’s control—he can’t
establish the conclusion. ask the patient to switch diseases for his
convenience—so (E) suggests the impossible.
(B) says that the evidence is impossible to challenge
in principle. But the argument uses historical records,
14. (E)
which can always be challenged in principle by other
records: documents showing discontent or rebellion in The nutritionist isn’t exactly crazy about the consumer
other sections of the pharaoh’s kingdom, for instance. advocate’s suggestion to publicize the disadvantages
of tropical oils; she worries that a campaign focusing
(D) The word “ancient” is only used once in the passage,
on tropical oils would do harm, because Americans
to describe Akhenaten, and it’s used in one sense only,
would make minor changes in their diet (cut down on
to mean that he lived a very long time ago. There’s
oil) and ignore more important changes (continue to
nothing ambiguous about this term.
wolf down meat, poultry etc.). So the focal point of the
(E) isn’t accurate; the argument is interested in what debate is the publicity campaign—the advocate
Akhenaten’s own people thought of him during his believes such a campaign would improve public health
reign, and used documents written by Akhenaten’s own by cutting consumption of tropical oils and hence of
contemporaries as evidence. The author himself saturated fat, whereas the nutritionist believes it will
doesn’t judge the pharaoh; he merely formulates an lead people to ignore the dangers posed by more
inappropriate conclusion based on the judgments of harmful sources of saturated fat, in which case public
the ancient palace guards. health would actually suffer. As (E) says, they disagree
over the wisdom of a campaign focusing on tropical
13. (D) oils.
The physician knows exactly three things: 1) a patient (A) The nutritionist never disputes that large quantities
is suffering from disease X or disease Y; 2) there’s no of tropical oil can cause heart disease; she only says
test that can distinguish disease X from disease Y; and North Americans are likely to get more saturated fats
finally, 3) there is an effective treatment for Y, but not from other sources.
for X. The physician concludes that the patient should
(B) The two debaters don’t disagree over whether the
be treated as if he has disease Y, and we’re asked to
publicity campaign can change the public’s behavior,
identify a principle on which that conclusion could be
they just disagree over whether the change caused by
based. It’s easy to follow the physician’s thinking: If
an anti-tropical oil campaign will be for the better.
the patient has disease X, no treatment will help; if he
has disease Y, the treatment for disease Y will help; so (C) is wrong because the consumer advocate never
with nothing to lose and everything to gain, we might compared the effects of reducing tropical oils to the
as well treat him for disease Y. (D) describes this effects of reducing meat; she merely said that it would
principle in general terms—when success is only be possible and beneficial to persuade people to
possible if circumstances-beyond-control are favorable reduce their tropical oil consumption. We have no clue
(the patient has the treatable disease Y), one should as to the advocate’s stance on meat.
act as if the circumstances are favorable (treat the (D) is similar to (A), focusing on an issue relating to
patient for disease Y). tropical oils that the nutritionist simply doesn’t touch.
(A) and (C) both fail to recognize that it’s impossible The nutritionist doesn’t deny that replacing tropical oils
to determine whether a patient has X or Y. Moreover, with healthier alternatives might help some people;
(A) recommends treating both diseases, while the she just doubted the wisdom of an ad campaign
physician can treat only one, Y. focusing only on tropical oils.
(B) Au-contraire! (B) says the physician should act as
15. (A)
if circumstances were unfavorable (as if the patient
has untreatable disease X), which is the opposite of The first step in this Parallel Reasoning question is to
what the physician recommends. abstract from the specifics of the stimulus. In general
terms, the stimulus provides a case in which a
statement uttered in response to a particular situation
32
Section IV: Logical Reasoning
is attacked as an ineffectual response; there are only (B) is correct in saying that Alicia attacks Concetta’s
two possible scenarios, and both expose the futility of evidence (the claim that Franchot was ahead of her
the statement (if the original position is wrong, the time) but totally misses the major focus of her
given response would make matters look worse; if the argument, which is a rejection of Concetta’s criterion
position is right; the response does nothing to prove it). for a great writer; moreover, Alicia doesn’t generalize
The parallel choice we’re looking for should therefore from new evidence.
show that a given response shouldn’t be used (D) Alicia never offers facts to support the notion that
because, whether the party under attack is right or the mark of a great writer is the ability to move people
wrong, the response is ineffective. We don’t have to with the written word. She does claim that the facts of
look far. (A) says that when a practice has been industrialization were well known at Franchot’s time,
denounced, it shouldn’t be defended by saying, “this is but this merely undermines Concetta’s claim, and
how we’ve always done it.” The reason for this is provides no support for her alternative criterion.
exactly parallel to the original’s “lose-lose” element: if
(E) ignores the fact that Alicia attacks two of Concetta’s
the practice is a poor one, the response makes
claims (that she was a great writer and ahead of her
matters look even worse; if the practice is wise, the
time) as well as her criterion for great writers. More
answer doesn’t provide good support for it. (A) fits the
importantly, Alicia never attacks the structure of
stimulus form to a T.
Concetta’s argument (there’s little structure to attack).
(B) Unlike the stimulus, there’s no issue of the initial
opinion being right or wrong. Instead of being 17. (B)
presented with two cases where a person who eats
Anson has taken umbrage at Dr. Ladlow’s conclusion
strange foods is wrong or right to do so, we’re given
that his theory is “irrefutably correct.” He argues that
two ways the response will sound unsatisfying.
Dr. Ladlow isn’t a responsible psychologist, because a
(C) also begins with a response people shouldn’t make, responsible psychologist would always realize that no
but instead of showing why the justification is pointless theory can be called irrefutable, since new evidence
in the cases of the person being right and being wrong, can always come to light. Answer choice (B) essentially
the argument attacks the response for avoiding the restates Anson’s attack on Dr. Ladlow in general
issue. terms. Dr. Ladlow didn’t fit Anson’s criterion of a
(D) again fails to show why the scholars’ response is no responsible psychologist because he didn’t recognize
good whether they are right or wrong, implying instead the possibility that contradictory evidence could refute
that they are probably partly right and partly wrong. his theory. Answer choice (B) says that a psychologist
(E) fails because it recommends two better ways of who obtains consistent results can’t responsibly
responding (admission of error or silence), whereas the conclude that his theor y is irrefutable. This is
stimulus was primarily interested in describing why the essentially the principle that Anson’s argument rests
original response was no good whether the original on, so answer choice (B) can be inferred from Anson’s
statement was right or wrong. argument.
(A) is a distortion of the argument; Anson doesn’t
16. (C) question the accuracy of Ladlow’s evidence, he just
Alicia’s argument is twofold. First she disagrees with points out the possibility (which Ladlow ignored) that
Concetta’s criterion for a great writer, which is that the new evidence might turn up.
person grasps a social issue ahead of her time (in this (C) is relatively tricky. Anson doesn’t say that
case industrialization); she offers instead her own psychologists can never be correct, only that they can
criterion, which is that the person has the ability to never be absolutely sure that they’re correct. For
move people through writing. Then she disagrees with instance, Ladlow’s theory may well be correct, but he
Concetta’s claim that Franchot was in fact one of the can’t responsibly claim to know, beyond a shadow of
first to recognize the effect of industrialization. This is doubt, that it’s correct.
the method that (C) describes: she rejects Concetta’s (D) has nothing to do with Anson’s argument. Anson
criterion of what makes a great writer, and disputes a isn’t questioning the content of Ladlow’s experiments,
specific claim (that Franchot was ahead of her time). only his conclusion that his theory is irrefutable.
(A) The first word tells you why (A) is wrong—Alicia (E) makes a familiar error. Anson says: “if a psychologist
doesn’t accept anything Concetta says, much less her is responsible, he admits his theory might be disproved.”
criterion of what makes a great writer. (E) states: “if a psychologist admits his theory might
33
PrepTest 4 Explained
be disproved, he must be responsible.” But there (B) The doctors are presented as experts, not simply
might be other requirements for a psychologist to be “typical cases,” and the author uses their behavior as
considered responsible. a guide precisely because they are experts.
(C) is wrong, because although Smith states his
18. (B) conclusion in the beginning, he isn’t assuming this
Again we’re asked to understand Anson’s argument, conclusion; he provides evidence, the behavior of the
this time in more general terms. On what does Anson doctors. The reasoning is flawed because it’s based on
base his conclusion about Dr. Ladlow? He says that a questionable assumption, but not because it’s
responsible psychologists always behave a certain circular.
way, and since Ladlow does not behave in that way, he (D) The only authority mentioned in the stimulus is the
is not a responsible psychologist. As (B) observes, he doctors and there’s no hint that doctors give conflicting
is taking a general principle (about responsible advice. Besides, the author bases his conclusion not
psychology) and applying it to the case of Dr. Ladlow. on doctors’ advice, but on their behavior.
(A) Anson doesn’t base his argument on an attack
against Ladlow’s character. His conclusion could be 20. (D)
seen to attack Ladlow’s character, but that conclusion The stimulus concludes that the rise in prosperity in
is based on an application of the principle that England after 1840 was caused by the free trade
responsible psychologists admit their fallibility. policy, because the economy improved only after the
(C) There’s simply no ambiguous term being used in policy was implemented. The author is assuming a
Anson’s argument; the term “responsible” is the only causal connection from a temporal correlation. The
candidate, and Anson tells us exactly what he means general form of the argument is: Y happened after X;
by it. therefore, X must have caused Y. We see that form in
(D) Anson never questions the factual validity of Dr. (D), because an improvement in a company’s
Ladlow’s evidence, that is, the facts about his rat profitability occurred after a morale-building program
predictions. As we saw in the previous question, Anson was instituted, that program must have caused the
doesn’t attack Ladlow’s actual theory, but Ladlow’s improvement. As in the stimulus, the author concludes
belief that his theory is irrefutable. causation based on a temporal correlation.
(E) Again, the theory itself is not the focus of Anson’s (A) concludes that since no marsh hawks were found
argument. Anson never discusses Ladlow’s theoretical in the marsh last year, none will be found this year; that
explanation of rat behavior. His point isn’t that the is, (A) reasons that past performance is a good
theory is wrong, but that it can’t be considered predictor of future performance. There’s no assumption
irrefutable. of causality, so (A)’s not parallel.
(B) The reason for concluding that a bypass road
19. (E) probably helped the flow of traffic is not a temporal
Smith tells us that meat must be healthy, because coincidence, as in the stimulus. Also, once we see the
most doctors eat meat, and hey, who knows more word “probably,” we know that this argument is way too
about health than doctors do? If doctors knew meat to qualified to be parallel to the original, which, along with
be unhealthy, he implies, they wouldn’t eat it. He’s correct choice (D), is not qualified at all: Both
assuming that doctors, because of their great arguments conclude that one thing caused another, not
knowledge concerning nutrition, actually use that that one thing probably caused another.
knowledge to guide their eating habits. As (E) says, (C) is simply the observation of an action’s effect. The
he’s assuming that the experts (in this case doctors) stimulus on the other hand, concluded that a certain
do not act contrary to what their expertise tells them is action must have been responsible for a certain effect,
in their best interest (i.e. do not eat unhealthily). That’s because the effect occurred after that action.
a questionable assumption, and the basis for the (E) is the opposite of the stimulus. (E) concludes that
flawed logic here. because the asteroid collision caused the extinction of
(A) On the contrary, the issue of the motives of dinosaurs, that extinction couldn’t have taken place
Smith’s opponents (the people who believe meat is before the collision; i.e. Since X caused Y, Y couldn’t
unhealthy) never arises. have happened before X. That’s just common sense.
34
Section IV: Logical Reasoning
35
PrepTest 4 Explained
(C) is out for basically the same reason; the author 25. (E)
makes no comparison between the brain size of twins The stimulus study found that when people drank
and people in general. The stimulus only compares alcoholic beverages with their meals, they consumed
identical twins, who should have identical-size brains, about 175 more calories, from NON-alcoholic sources
to each other. than when they ate meals without consuming alcoholic
(D) sounds interesting, but it needn’t be true. beverages. In other words, when people are pounding
Schizophrenia could, in accordance with the author’s down the booze, they tend to be eating more, or at
theory, reduce brain size, but it could do so by different least consuming more food calories, than they do when
amounts in different twins, especially if their brains they aren’t drinking. All of the choices help explain why
suffer structural damage of differing severity. this is so … all except correct choice (E). (E) does
(E) No; schizophrenia could be more likely to develop present a difference between the two types of meals
in identical twins than in the population at large and (alcoholic and alcohol-free), but it’s not a relevant
this would have no effect on the conclusion. difference. The issue isn’t what kind of calories were
consumed, but how many. In breaking down the two
24. (D) types of meals into their proportional sources of
calories, (E) does nothing to explain why the total
Now we’re looking for a statement that contradicts the
amount of calories in the two meal situations differ.
stimulus evidence. The stimulus tells us that the brains
of identical twins are “genetically identical.” Thus, their (A) If people linger at the table when they consume
brains contain the same genetic information; there are alcohol, it’s not surprising that they spend a lot of that
no genetic differences between them. So, in the cases extra time eating, and so tend to consume more
discussed, it would not be possible to determine from calories.
genetic information alone, that the one twin will (B) If the alcoholic meals, since they occurred later in
develop schizophrenia and the other won’t. Therefore, the day, were larger meals, then it’s no surprise at all
genetic information alone isn’t enough to indicate that people consumed more calories at those bigger
whether a person will develop schizophrenia. If the meals.
statements in the stimulus are to be believed, (D) is (C) says that people eat more when there are a lot of
flat out wrong, and therefore the answer we seek. them eating together, and that alcohol tends to be
(A) could be true; genetic susceptibility to schizophrenia served at meals that have a lot of diners. So here too
could be one of its prerequisites. Considering the we have an explanation: The large number of diners,
evidence we’re given, it’s possible that a pair of which correlates well with the serving of alcohol,
identical twins share a genetic susceptibility to results in greater food consumption.
schizophrenia, but that only one actually develops the (D) If meals at which alcohol is served tend to be more
disease (for whatever reason). “enticing” in both preparation and appearance, then
(B) Nothing in the stimulus refers to the treatment of it’s understandable that diners may be enticed to eat
schizophrenia, so there’s nothing to contradict (B)’s more of those meals.
claim.
(C) The stimulus never says that the brains of
schizophrenic twins are completely different from the
brains of healthy twins, only that certain areas of the
brain are smaller. The brains of schizophrenics might
share many characteristics with the brains of people
without the disorder.
(E) provides a cause—viral infections—for the brain
abnormalities associated with schizophrenia. This fits
the stimulus rather well: maybe viral infections damage
the brain and the damage causes schizophrenia. No
contradiction.
36
1-800-KAP-TEST | kaptest.com
ÖLL3124Aqä
LL3124A
*LSAT is a registered trademark of the Law School Admission Council. Printed in USA ©2008 Kaplan, Inc.